PT7.S4.Q11 - weapons research

Having a bit of trouble understanding why A is correct (I originally chose C). Could someone clarify? Thank you!

Stim:
Ctx: Foundation didn't want their $ to be used for weapons research so uni said none of their $ would be used for the research
P: None of foundation's $ would benefit weapons research
C: Foundation rescinded threat

ACs:
C - I didn't love this AC, but chose it because none of the others initially seemed correct to me. Descriptively, it is accurate in that the foundation overlooked this possibility that the uni was lying, but I mean it's a short argument, I'm sure they could've overlooked several things.

I wonder if this C almost attacks the premise in a way - Like, okay sure what if the uni was lying? Then the foundation's grant money would DIRECTLY benefit weapons research. Is this the right way of thinking about it?

A - I googled around and saw some explanations for why A was correct, but my struggle is that the crux of why this AC is correct is that it hinges on the nuance of direct vs indirect benefit. I didn't even think about indirect benefit until reading explanations for this AC, but wondering if that's a common theme of "direct" vs "indirect" when it comes to certain flaw questions?

help

Comments

  • LivinLaVidaLSATLivinLaVidaLSAT Alum Member
    705 karma

    I mean it's a short argument, I'm sure they could've overlooked several things.

    You’re right. Have this thought with every stimulus. That’s why LR isn’t full of solid arguments- words are limited. One of my big takeaways from The Loophole book is not to give the author credit for anything that wasn’t said.

    C: I think you misunderstood C. You make the leap that this answer means they were lying. I wouldn’t equate this to lying because that’s not what it says. Did C happen in the argument? Yes. As you said, it's descriptively accurate. Is it a flaw? No. The uni didn’t miss the promise they made. It’s their reason for withdrawing the threat.

    “Overlooks the possibility” means something the author didn’t think about in reaching their conclusion and for it to be a flaw, the statement that follows that opener must weaken the conclusion (it causes a problem for the argument). Test this strategy with answers A, B, and C.

    A: This answer addresses the term shift that shows a faulty generalization in the argument. That was the flaw I saw. The uni is equating not using the money with not benefiting. They overgeneralized in the conclusion. I’m not sure if “direct” vs. “indirect” is a theme. That doesn’t stick out in my memory (haven’t studied in awhile).

    I hope that helps.

  • moonstars5678moonstars5678 Member
    166 karma

    @LivinLaVidaLSAT - thank you, that's definitely helpful. So to expand on the "overlook the possibility" clarification you made: Overlooking the possibility in itself isn't always the problem (since being descriptively accurate isn't what makes the AC true). What makes the AC true is that what they overlook is what really creates a flaw in the argument (i.e., for A, if the uni could redirect funds from the humanitarian use to weapons research, then the foundation's conclusion is weakened.)

  • LivinLaVidaLSATLivinLaVidaLSAT Alum Member
    705 karma

    Right! For a flaw question, when looking at the answers, ask:

    1. Did this happen?
    2. Is this a problem for the argument?

    The answer must be yes to both for it to be a flaw.

Sign In or Register to comment.